Integration by Parts: Solving \int64x^2cos(4x)dx

Click For Summary
SUMMARY

The integration of the function \(\int 64x^2 \cos(4x) \, dx\) can be effectively solved using the method of integration by parts. The recommended approach involves a substitution of \(4x\) with \(u\), simplifying the integration process. After the substitution, the integration by parts should be applied twice to arrive at the correct solution. This method ensures clarity and reduces the complexity of the resulting expressions.

PREREQUISITES
  • Understanding of integration by parts
  • Familiarity with trigonometric functions, specifically cosine
  • Knowledge of substitution techniques in calculus
  • Basic algebraic manipulation skills
NEXT STEPS
  • Practice integration by parts with different functions
  • Explore substitution methods in calculus
  • Review trigonometric identities and their applications in integration
  • Study advanced integration techniques, including repeated integration by parts
USEFUL FOR

Students and educators in calculus, mathematicians focusing on integration techniques, and anyone looking to enhance their problem-solving skills in integral calculus.

chrono210
Messages
6
Reaction score
0
How would you go about doing this:

\int64x^2cos(4x)dx

The question specifically asks to integrate it by parts, so I integrated it that way a couple of times and came out with some long mess of sines and cosines, but it's not the right answer.

Thanks.
 
Physics news on Phys.org
The approach is correct. The first thing I'd do is a substitution 4x->u just so that the numbers disappear. Then just integrate by parts twice. I suggest you try and post your attempt, so that we can see if it's just a simple algebraic mistake. Like I said, it's going in the right direction.
 
Oops, I almost forgot about this. I actually was able to figure it out. Thanks though. :)
 
Question: A clock's minute hand has length 4 and its hour hand has length 3. What is the distance between the tips at the moment when it is increasing most rapidly?(Putnam Exam Question) Answer: Making assumption that both the hands moves at constant angular velocities, the answer is ## \sqrt{7} .## But don't you think this assumption is somewhat doubtful and wrong?

Similar threads

  • · Replies 3 ·
Replies
3
Views
2K
  • · Replies 6 ·
Replies
6
Views
2K
  • · Replies 2 ·
Replies
2
Views
2K
Replies
3
Views
2K
  • · Replies 3 ·
Replies
3
Views
5K
  • · Replies 22 ·
Replies
22
Views
3K
  • · Replies 13 ·
Replies
13
Views
2K
Replies
6
Views
2K
  • · Replies 4 ·
Replies
4
Views
3K
  • · Replies 2 ·
Replies
2
Views
2K